-5
-4
-3
-2
- 1
-3
-4
-5
The equation of the line in this graph is y=
2021 Edmentum. All rights reserved.

-5-4-3-2- 1-3-4-5The Equation Of The Line In This Graph Is Y=2021 Edmentum. All Rights Reserved.

Answers

Answer 1

weThe above question is a linear equation, we can find the equation by doing the following.

1) Determine which type of equation, linear or exponential, will fit the graph.

2) Identify at least three points from the graph.

3) Find the slope of the line, using any two of the points.

4) Find the Y-intercept of the line.

5) Use the slope and Y -intercept to find an equation of the line.

STEP 1

we can identify the following points

\(\begin{gathered} x_1=0_{}_{} \\ x_2\text{ = -2} \\ y_1=0_{} \\ y_2\text{ = - 3} \end{gathered}\)

STEP 2

we then find the slope,

\(\text{slope(m) = }\frac{y_2-y_1}{x_2-x_1}\text{ = }\frac{-3-0}{-2-0}\text{ =}\frac{-3}{-2}=\frac{3}{2}\)

STEP 3

We identify the y-intercept as the point at which the line crosses the y axis.

Y - Intercept(c) = 3

STEP 4

We substitute the value of the slope and the y-intercept into the formula for the equation of a line below.

\(y=\text{ mx + c (equation of a line)}\)\(\begin{gathered} y\text{ =}\frac{3}{2}x\text{ + 3} \\ we\text{ multiply through by the lcm of the denominator which is 2} \\ 2\text{ x y = 2 x }\frac{3}{2}x\text{ + 2 x 3} \\ 2y\text{ = 3x + 6} \end{gathered}\)

Therefore, the equation of the line in the graph is 2y =3x + 6


Related Questions

The number of animal in a park increase by 60% every month. By the end on December the number of animals is 2560, how many animals would have been there by the end of November?

Answers

Answer:

1,600

Step-by-step explanation:

I did a bit of guessing becuase I don't know the exact way to reverse the formula, but here is what I did!

I guessed around 1,550 at first, but it was too low. I kept increasing the base number until I reached 1,600, and came to the conclusion by doing the equation;

1,600(0.6)=960

Then, since it increases each month by 60% of the previous month's value, you do

1,600+960=2,560

PLEASE HELP IM TAKING THE TEST NOW AND I CANT FIGURE IT OUT PLEASE HELP!!!!!!!!!!!!!! Choose the function table that matches the given rule. (1 point)
Rule: output = input - 6
o
Input Output
5
-2
3
-6
0
-6
O Input Output
- 7
-13
-2
-8
6
0
Input
Output
9
3
-6
2
6
0
Input
Output
-2.
-8
2
8
5
-3

Answers

Answer:

what's

Step-by-step explanation:

the test

years
0
2
4
5
9
cost

Answers

Answer:

what is this what does this mean

Step-by-step explanation:

If c>0 , |u| > c is equivalent to u < or u>

If c&gt;0 , |u| &gt; c is equivalent to u &lt; or u&gt;

Answers

The true statement is now presented: If \(c > 0\) and \(|u| > c\), then \(u > c\) or \(u < -c\).

Mathematically speaking, the absolute value is defined by following expression:

\(|a| = \left \{ {{a,\,a \ge 0} \atop {-a,\,a < 0}} \right.\) (1)

If \(c > 0\) and \(|u| > c\), then \(u > c\) or \(-u > c\). Hence, \(u > c\) or \(u < -c\).

Therefore, if \(c > 0\) and \(|u| > c\), then \(u > c\) or \(u < -c\). (By tricotomy)

We kindly invite to see this question on absolute values: https://brainly.com/question/4691050

A basketball player made 80 out of 100 attempted free throws what percent of free throws was made ?

Answers

Answer:

8/10

Step-by-step explanation:

id hope your have a great day

Answer:

80%

Step-by-step explanation:

80÷100=.8 which is 80%

F(x)=x2-2x find f(x2)

Answers

answer

0

Step-by-step explanation:

you plug in the 2 into every xbox value

Answer:

f(x)= x²-2x

For finding f(x²), replace x with x²

f(x²)= (x²)²-2x²

= x⁴-2x²

Hope it helps :-)

Please solve this
∫ (log(1 + x ^ 2))/((x + 1) ^ 2) dx

Answers

The final result of the integral is:

∫ (log(1 + x²) / (x + 1)²) dx = log(x + 1) - 2 (log(x + 1) / x) - 2Li(x) + C,

where Li(x) is the logarithmic integral function and C is the constant of integration.

We have,

To solve the integral ∫ (log(1 + x²) / (x + 1)²) dx, we can use the method of substitution.

Let's substitute u = x + 1, which implies du = dx. Making this substitution, the integral becomes:

∫ (log(1 + (u-1)²) / u²) du.

Expanding the numerator, we have:

∫ (log(1 + u² - 2u + 1) / u²) du

= ∫ (log(u² - 2u + 2) / u²) du.

Now, let's split the logarithm using the properties of logarithms:

∫ (log(u² - 2u + 2) - log(u²)) / u² du

= ∫ (log(u² - 2u + 2) / u²) du - ∫ (log(u²) / u²) du.

We can simplify the second integral:

∫ (log(u²) / u²) du = ∫ (2 log(u) / u²) du.

Using the power rule for integration, we can integrate both terms:

∫ (log(u² - 2u + 2) / u²) du = log(u² - 2u + 2) / u - 2 ∫ (log(u) / u³) du.

Now, let's focus on the second integral:

∫ (log(u) / u³) du.

This integral does not have a simple closed-form solution in terms of elementary functions.

It can be expressed in terms of a special function called the logarithmic integral, denoted as Li(x).

Therefore,

The final result of the integral is:

∫ (log(1 + x²) / (x + 1)²) dx = log(x + 1) - 2 (log(x + 1) / x) - 2Li(x) + C,

where Li(x) is the logarithmic integral function and C is the constant of integration.

Learn more about integrations here:

https://brainly.com/question/30217024

#SPJ1

Find the slope of this problem

Find the slope of this problem

Answers

Answer:

1/9 rise 1 run 9

Step-by-step explanation:

if you graph the y coordinate as 3 and find the x value, x value should be 0. Point (0,3) and (-9,2) and do rise over run

the heigh of a rocket, h, is increasing at a constant rate of 18 feet per second. If it's height at five seconds is 118 feet, then write a linear equation for h as a function of time, t, in seconds since it was fired

Answers

A linear equation for h as a function of time, t, in seconds since it was fired is; h(t) =  (18t + 28) ft

How to find a Projectile equation?

We are given that the Rate of increasing of height is 18 ft/s

We are told that at t = 5 seconds, the height is 118 feet,

This means that;

h(5) = 118 ft

We know that height of rocket is increasing 18 ft for every second passed, and it will also have an initial height.

Thus;

h(t) = 18t + h₀

where;

h₀ is the initial height

Plugging in the relevant values for a height of 5 seconds gives;

118 = (18 * 5) + h₀

h₀ = 118 -90

h₀ = 28 ft

Thus, the equation for the height is;

h(t) =  (18t + 28) ft at a time t seconds.

Read more about projectile equation at; https://brainly.com/question/16866199

#SPJ1

The amount of money deposited into an account in a given year is multiplied by a growth factor of x = 1 + r. $800 is
deposited at the start of the first year; an additional $300 is deposited at the start of the next year, and $625 at the start of
the following year. Write an expression that represents the amount of money in the account at the end of the three years.

Answers

Answer:

800(1 + r)³ + 300(1 + r)² + 625(1 + r)

Step-by-step explanation:

At the beginning of the first year $800 is deposited.

At the end of the first year the amount of money in account = $800 × growth factor = 800(1 + r)

At the beginning of the second year, $300 is deposited, the money in account = 800(1 + r) + 300

At the end of the second year the amount of money in account is multiplied by growth factor. = [800(1 + r) + 300] × 1 + r = 800(1 + r)² + 300(1 + r)

At the beginning of the third year, $625 is deposited, the money in account = 800(1 + r)² + 300(1 + r) + 625

At the end of third year the money = [800(1 + r)² + 300(1 + r) + 625] × 1 + r = 800(1 + r)³ + 300(1 + r)² + 625(1 + r)

I need help
I need help
I need help
I need help
I need help
I need help
I need help

I need helpI need helpI need helpI need helpI need helpI need helpI need help

Answers

The sequence is decreasing as n increases and sequence converges to the value 0.

The given sequence is defined as aₙ = 1 / (7n + 3).

To determine if the sequence converges or diverges, we need to analyze its behavior as n approaches infinity.

As n increases, the denominator 7n + 3 also increases which means that the values of aₙ will get smaller and smaller, approaching zero as n becomes larger.

The sequence converges to the value 0.

The sequence is decreasing as n increases.

The sequence converges to the value 0.

To learn more on Sequence click:

https://brainly.com/question/21961097

#SPJ1

in problems 21 through 30, first verify that the given vectors are solutions of the given system. then use the wronskian to show that they are linearly independent. finally, write the general solution of the system. 25

Answers

It is verified that the given vectors x₁ and x₂ are solutions of the given system. Using Wronskian it is shown that they are linearly independent. Then, \(W(t)=7e^{-3t}\). The general solution of the given system is written as \(x(t)=\left[\begin{array}{c}3c_1e^{2t}+c_2e^{-5t}\\2c_1e^{2t}+3c_2e^{-5t}\end{array}\right]\).

Wronskian analysis helps to determine whether a solution is linearly dependent or independent.

Given the system is \(x'=\left[\begin{array}{ccc}4&-3\\6&-7\end{array}\right]x\).

Let's differentiate x₁ concerning t, and we get,

\(\begin{aligned}x_1'&=\left[\begin{array}{c}\frac{d}{dt}(3e^{2t})&\\\frac{d}{dt}(2e^{2t})&\end{array}\right] \\&=\left[\begin{array}{c}6e^{2t}&\\4e^{2t}&\end{array}\right] \end{aligned}\)

Now,

\(\begin{aligned}\left[\begin{array}{ccc}4&-3\\6&-7\end{array}\right]x_1&=\left[\begin{array}{ccc}4&-3\\6&-7\end{array}\right]\left[\begin{array}{c}3e^{2t}\\2e^{2t}\end{array}\right]\\&=\left[\begin{array}{c}12e^{2t}-6e^{2t}\\18e^{2t}-14e^{2t}\end{array}\right]\\&=\left[\begin{array}{c}6e^{2t}\\4e^{2t}\end{array}\right]\\&=x_1'\end{aligned}\)

From this, we can write,

\(x_1'=\left[\begin{array}{cc}4&-3\\6&-7\end{array}\right]x_1\)

Therefore, we conclude that x₁ is a solution to the given system.

Let's differentiate x₂ concerning t, and we get,

\(\begin{aligned}x_2'&=\left[\begin{array}{c}\frac{d}{dt}(e^{-5t})&\\\frac{d}{dt}(3e^{-5t})&\end{array}\right] \\&=\left[\begin{array}{c}-5e^{-5t}&\\-15e^{-5t}&\end{array}\right] \end{aligned}\)

Now,

\(\begin{aligned}\left[\begin{array}{ccc}4&-3\\6&-7\end{array}\right]x_2&=\left[\begin{array}{ccc}4&-3\\6&-7\end{array}\right]\left[\begin{array}{c}e^{-5t}\\3e^{-5t}\end{array}\right]\\&=\left[\begin{array}{c}4e^{-5t}-9e^{-5t}\\6e^{-5t}-21e^{-5t}\end{array}\right]\\&=\left[\begin{array}{c}-5e^{-5t}\\-15e^{-5t}\end{array}\right]\\&=x_2'\end{aligned}\)

From this, we can write,

\(x_2'=\left[\begin{array}{cc}4&-3\\6&-7\end{array}\right]x_2\)

Therefore, we conclude that x₂ is also a solution to the given system.

Now, find the Wronskian of x₁ and x₂, we get,

\(\begin{aligned}W(t)&=\text{det}[x_1\;x_2]\\&=\text{det}\left[\begin{array}{cc}3e^{2t}&e^{-5t}\\2e^{2t}&3e^{-5t}\end{array}\right] \\&=(3e^{2t}\times3e^{-5t})-(e^{-5t}\times2e^{2t})\\&=9e^{2t-5t}-2e^{2t-5t}\\&=9e^{-3t}-2e^{-3t}\\&=7e^{-3t}\\&\neq0\end{aligned}\)

From this, we can conclude that x₁ and x₂ are independent.

Finally, we write the general solution of the system as follows,

\(\begin{aligned}x(t)&=c_1x_1+c_2x_2\\&=c_1 \left[\begin{array}{c}3e^{2t}\\2e^{2t}\end{array}\right] +c_2\left[\begin{array}{c}e^{-5t}\\3e^{-5t}\end{array}\right] \\&=\left[\begin{array}{c}3c_1e^{2t}\\2c_1e^{2t}\end{array}\right] +\left[\begin{array}{c}c_2e^{-5t}\\3c_2e^{-5t}\end{array}\right]\\&=\left[\begin{array}{c}3c_1e^{2t}+c_2e^{-5t}\\2c_1e^{2t}+3c_2e^{-5t}\end{array}\right] \end{aligned}\)

The complete question is -

First, verify that the given vectors are solutions of the given system. Then use the Wronskian to show that they are linearly independent. Finally, write the general solution of the system.

\(x'=\left[\begin{array}{ccc}4&-3\\6&-7\end{array}\right]x;\; x_1=\left[\begin{array}{ccc}3e^{2t}\\2e^{2t}\end{array}\right], \;x_2=\left[\begin{array}{cc}e^{-5t}\\3e^{-5t}\end{array}\right]\)

To know more about Wronskian:

https://brainly.com/question/14309076

#SPJ4

During which two moon phases would the amount of light reflected from the moon appear to be equal from Earth? Why does this occur at different times in a month?

A.
The light reflected appears the same during the new moon and full moon phases because the sun, moon, and Earth are aligned.

B.
The light reflected appears the same during the new moon and first quarter moon phases because the sun, moon, and Earth are at the same angles.

C.
The light reflected appears the same during the full moon and third quarter moon phases because the sun, moon, and Earth are at the same angles.

D.
The light reflected during the first quarter and third quarter moon phases appears equal because the sun, moon, and Earth are at right angles.

Answers

Answer:

D

The light reflected during the first quarter and third quarter moon phases appears equal because the sun, moon, and Earth are at right angles.

If the discriminant of an equation is zero, which of the following is true of the
equation?
A. It has two complex solutions.
B. It has two real solutions.
C. It has one complex solution.
D. It has one real solution.
SUBMI

Answers

Answer:

Step-by-step explanation:

if the discriminate is 0, it means that both roots are the same. Not only that, but it also means that the roots are real. I would pick D, but realize that that is the expected answer and the answer could be B, depending on how the person writing the problem thinks about it.

9.
(03.05 LC)
The functions f(x) and g(x) are described below:
f(x) = 3x − 7
g(x) = 3x + 8
The graph of g(x) is obtained by shifting up the graph of f(x) by

units. (5 points)

Answers

♥️♥️♥️♥️♥️♥️♥️♥️♥️♥️♥️♥️

The graph of g(x) is obtained by shifting up the graph of f(x) by 15 units.

♥️♥️♥️♥️♥️♥️♥️♥️♥️♥️♥️♥️

I NEED HELPP Solve for PMR please

I NEED HELPP Solve for PMR please

Answers

The angle PMR in the quadrilateral is 32 degrees.

How to find the angle PMR?

The angle PMR can be found as follows;

The line AP is an angle bisector of angle RPM. Therefore, the following relationships are formed.

∠RPM ≅ ∠WPM

Hence,

∠RPM ≅ ∠WPM = 58 degrees

Therefore,

∠WPM = 58 degrees

∠PWM = 90 degrees

Let's find ∠PMR as follows

∠PMR = 180 - 90 - 58

∠PMR = 90 - 58

∠PMR  = 32 degrees

learn more on angles here:https://brainly.com/question/25779160

#SPJ1

The data set includes home values (in ten thousands of dollars) for all the homes in a small neighborhood. {7.39, 8.69,9.28, 9.66, 10.20, 10.84, 12.15, 36.44} Find the mean of the home. (Round to the nearest tenth)

Answers

Answer: 13.1

Explanation:

The data set we have is:

\(\left\{ 7.39,8.69,9.28,9.66,10.20,10.84,12.15,36.44\right\} \)

To calculate the mean, we need to add all the quantities and divide by the number of elements in the data set:

\(Mean=\frac{sum\text{ of all the data}}{number\text{ of elements}}\)

The number of elements in the data set is: 8 elements

And the sum of all the data is:

\(7.39+8.69+9.28+9.66+10.20+10.84+12.15+36.44=104.65\)

So, we substitute this into the Mean formula, and get the following result:

\(\begin{gathered} Mean=\frac{104.65}{8} \\ Mean=13.081 \end{gathered}\)

Rounding to the nearest tenth (1 decimal place):

\(Mean=13.1\)

Answer: 13.1

Lorraine prepared a 2.5-gallon pot filled with tomatoes to be canned in jars. Each jar will hold 1.25 quarts of tomatoes. If 1 gallon equals 4 quarts, how many jars can Lorraine fill?

4 jars
5 jars
8 jars
10 jars

Answers

Answer:

  (c)  8 jars

Step-by-step explanation:

You want to know the number of 1.25 quart jars that can be filled from a 2.5 gallon container.

Jars

The number of jars can be found from ...

  (2.5 gal) × (4 qt/gal) × (1 jar)/(1.25 qt) = (2.5·4/1.25) jars = 8 jars

Lorraine can fill 8 jars from the 2.5 gallon pot.

<95141404393>

Write the equation
(x-3)^2+(y-5)^2=25 in general form

Answers

x²+6x+y²-10y+9=0

Answer:

Solution given:

(x-3)^2+(y-5)^2=25

Using formula (a-b)²=a²-2ab+b²

x²-2*x*3+3²+y²-2*y*5+5²=25

x²+6x+9+y²-10y+25=25

x²+6x+y²-10y+9=25-25

x²+6x+y²-10y+9=0 is a general form.

The length of ribbons found at a seamstress are listed.

3, 11, 11, 13, 13, 21

What is the appropriate measure of variability for the data shown, and what is its value?

The mean is the best measure of variability and equals 11.
The median is the best measure of variability and equals 11.5.
The range is the best measure of variability and equals 18.
The IQR is the best measure of variability and equals 2.

Answers

The appropriate measure of variability for the given data is the range, which is the difference between the maximum and minimum values in the dataset. In this case, the range is calculated as 21 - 3 = 18. The range provides a straightforward and intuitive measure of how spread out the data is. The other options (mean, median, and IQR) are measures of central tendency or spread and may not capture the full range of variability in this dataset. Therefore, the correct answer is C. The range is the best measure of variability, and its value is 18.

What percentage of values are between 25 and 45

What percentage of values are between 25 and 45

Answers

Answer:

I'm not that good at this but I'll try.

The difference between 25 and 40 is 15

The difference between 80 and 5 is 75

So, 15/75 = 0.20 or 20%

Sorry, that all I can come up with

Step-by-step explanation:

The temperature of an enclosure for a pet snake should be about 80°F, give or take 5°F. What are the maximum and minimum temperatures of the enclosure?

Answers

Answer:

The max is 85F

The min is 75F

Step-by-step explanation:

For the max 80+5=85F

For the min 80-5=75F

Max =85


Minimal=75


Your subtracting 5 from 80 and 85

factorise x³-4x²+x+6​

Answers

The binomial factors of x³- 4x²+x+6​ are (x+2), (x+3), and (x-1).

Using the splitting and grouping the terms:

x³ + 4x² + x - 6

= x³ + 2x² + 2x² + x - 6  [Splitting 4x² = 2x² + 2x²]

= (x³ + 2x²) + (2x² + x - 6)

= x² (x + 2) + (2x² + 4x - 3x - 6)

= x² (x + 2) + [ 2x (x + 2) - 3 (x + 2)]

= x² (x + 2) + (x + 2) (2x - 3)

= (x + 2) ( x² + 2x - 3)

= (x + 2) ( x² + 3x - x - 3)

= (x + 2) [x (x + 3) - 1 (x + 3)]

= (x + 2) (x + 3) (x - 1)

Hence, the binomial factors are (x + 2), (x + 3) and (x - 1)

To learn more about factorise here,

https://brainly.com/question/10718512

https://brainly.com/question/24734894

e the function.
R
ƒ(x) = −x² – 10x + 16
Find f(-7)
Submit Answer

Answers

Answer

37

Step by Step explanation

replace x with -7

which gives

-(-7)^2-10(-7)+16

= 37

Suppose you model a game of chance with a discrete probability distribution. Let X be the net amount of money won or lost by the player. Let P ( X ) be the probability of the corresponding outcome. The three events are as follows: There is a 23% chance the player wins 5 dollars. There is a 29% chance the player breaks even. There is a 48% chance the player loses 3 dollars. Complete the table below to model the scenario

Answers

Mathematicians have used probability to determine how likely certain events are to occur. The possible values of  X will be 10, 0, -5 with following probabilities:

P(X = 10 ) = 0.23

P(X = 0 ) = 0.48

P(X = -5) = 0.29

What in mathematics is probability?

Probability is the ability to happen. . From 0 to 1 is used to express the value. Whenever we are unsure of how an event will turn out, we can talk about the probabilities of various outcomes, or how likely they are. The study of probability-based events is often known as statistics. The amount of favorable outcomes and the overall number of outcomes thus affect how likely an event is to occur. The probability is typically expressed as a ratio between the number of positive outcomes and all of the outcomes in the sample space.

Given:

The probability distribution of X can be represented as:

X      P(X=x)

-5      0.29

0       0.48

10      0.23

The outcomes is attached as table below.

To learn more about probability, visit:

https://brainly.com/question/28045837

#SPJ1

The complete question is:

The table below to model the scenario is attached below:

Suppose you model a game of chance with a discrete probability distribution. Let X be the net amount
Suppose you model a game of chance with a discrete probability distribution. Let X be the net amount

Answer with Solution
Answer with Solution
Answer with Solution
Answer with Solution
Answer with Solution​

Answer with SolutionAnswer with SolutionAnswer with SolutionAnswer with SolutionAnswer with Solution

Answers

Answer:

-5

Step-by-step explanation:

-3x + 7 < 11

-3(-5) + 7 < 11

Multiplying two negatives together, makes a positive number.

15 + 7 < 11

22 < 11

22 less than 11 is false, therefore -5 is the correct answer.

MATHEMATICS // ANSWER

The correct answer is 4

Here's the Solution:

- 3x + 7 ‹ 11

Transpose 7 to the right side

- 3x ‹ 11 - 7

Now subtract 11 and 7

- 3x ‹ 4

HOPE THIS HELPS. GOODLUCK:)

A basket of fruits contains 5 bananas, 8 mangoes, and 7 apples. Two fruits are randomly selected without replacement. Calculate the probability of selecting an apple and a banana.
Answers:
7/80
21/190
3/5
7/76

Answers

The probability of selecting an apple and a banana from the basket is 7/76.

To calculate the probability of selecting an apple and a banana from the basket of fruits, we need to determine the total number of possible outcomes and the number of favorable outcomes.

The total number of fruits in the basket is 5 bananas + 8 mangoes + 7 apples = 20 fruits.

When selecting the first fruit, there are 20 options. Let's say we select an apple. After removing one apple from the basket, there are 19 fruits left.

When selecting the second fruit, there are 19 options. This time, we want to select a banana. Since there are 5 bananas left in the basket, there are 5 favorable outcomes.

To calculate the probability, we divide the number of favorable outcomes by the total number of possible outcomes:

Probability = Number of favorable outcomes / Total number of possible outcomes = (Number of apples) * (Number of bananas) / (Total number of fruits) * (Total number of fruits - 1) = 7/20 * 5/19  = 35/380 = 7/76

For more such questions on probability

https://brainly.com/question/25870256

#SPJ8

5 Find the slope and the y-intercept represented by the equation y=x-4y=x−4

Answers

Answer:

Slope 1; Y-intercept -1

Step-by-step explanation:

The equation is written in the form of y=mx=b (slope-intercept) where m is the slope and b the y-intercept, then find the terms.

Find the circumference of the circle. Use =3.14

Find the circumference of the circle. Use =3.14

Answers

Answer:

50.24

Step-by-step explanation:

8 x 3.14 x 2 = 50.24

Consider a tree T with n vertices, where n is an odd integer greater than or equal to 3. Let v be a vertex of T. Prove that there exists a vertex u in T such that the distance between u and v is at most (n-1)/2

Answers

There must exist a vertex u in T such that the distance between u and v is at most (n-1)/2.

To prove the existence of a vertex u in tree T such that the distance between u and v is at most (n-1)/2, we can employ a contradiction argument. Assume that such a vertex u does not exist.

Since the number of vertices in T is odd, there must be at least one path from v to another vertex w such that the distance between v and w is greater than (n-1)/2.

Denote this path as P. Let x be the vertex on path P that is closest to v.

By assumption, the distance from x to v is greater than (n-1)/2. However, the remaining vertices on path P, excluding x, must have distances at least (n+1)/2 from v.

Therefore, the total number of vertices in T would be at least n + (n+1)/2 > n, which is a contradiction.

Hence, there must exist a vertex u in T such that the distance between u and v is at most (n-1)/2.

For more such questions on vertex

https://brainly.com/question/25651698

#SPJ8

Other Questions
X 4 and X>-2Interval Notation true or false. the arrow labeled C represents a transfer of chemical energy to mechanical energy. explain why this is true or false.true or false. the arrow labeled A represents a transfer of solar energy to chemical energy. explain why this is true or false. Jack and Shelly have saved up a total of $29.00. Jack has saved 4 dollars more than 23 as much as Shelly. How much has Shelly saved? is the degree to which carrying out the job's required activities results in the individual's obtaining direct and clear information about the effectiveness of his or her performance is called? explain the relationship between an atoms number of electrons and its relative stability A 30.0-g object moving to the right at 20.5 cm/s overtakes and collides elastically with a 13.0-g object moving in the same direction at 15.0 cm/s. Find the velocity of each object after the collision. (Take the positive direction to be to the right. Indicate the direction with the sign of your answer.) You want to transport 140 000 tons of granulate from DUQM to SOHAR The product has a S.G. of 0,4 The internal measures of the 30ft containers are:Length: 29'7" Width: 8'4" Height: 9'7" Occupation degree is 90% Weight of the container is 3 tons. Max. Payload of the container is 33 tons. Max. Weight of the train is 1600 tons. Length of the train is not relevant. We will use 4-axle SGNS wagons with a tare of 20 tons each. The capacity of a SGNS wagon is 60ft. a) How many containers do we have to transport? b) How many containers fit on a train? c) How many trains do we have to run? d) Debate the pros and cons of rail and road transport Jared works as a landscaper. he installs a sprinkler that sprays water in a circle with an 8-foot radius. what is the approximate area covered by the sprinkler? a 28-year-old woman presents to her primary care provider complaining of abdominal bloating, cramping and diarrhea most days of the week for the last four months. the diarrhea commonly occurs in the morning or following meals. episodes occur four or five times per day and are of moderate volume. she denies fever, nausea, vomiting, periods of constipation, or change in appetite. the diarrhea does not wake her from her sleep and she reports no blood or pus in her stool. she states the periods of diarrhea will last several months and then seem to resolve without explanation. in between diarrhea episodes, she has normal formed bowel movements once per day. a review of her symptom diary reveals no food associations. she denies any recent travel. what is the most likely diagnosis? Julia wants to read a book that has 176 pages. She reads 95 pages one week. For the next 9 days, she will read the same number of pages each dayuntil she finishes reading the book.How many pages should julia read each day to finish the book? Enter the answer in the box.pages 12. The first Presidential election that women were allowed to vote in was won by _____. the interest on a 6%, 60-day note for $5,000 is $300. true or false The value perspective of quality is most important at the FILL IN THE BLANK stage in the creation and delivery of goods and services. A circle has a circumference of 16 yards. Find the area in terms of . Given the following linear system: X1 4x2 + 2x3 = 33x2 + 5x3 = -7-2x1 = 8x2 4x3 = -3Is this system consistent? Yes, this system is consistent, No, this system is inconsistent. Which of the following are examples of musical parameters? Select ALL thatapplyA. TempoO B. FragmentationO C. ArticulationD. Dynamics When sexology originated in the 19th century, it promoted the __________ model of sexuality. Read the passage.The man at the gas station gave me directions and then kept on chatting, telling me about the weather to come and places to stay nearby. His laconic wife simply nodded her head, occasionally correcting him on his lefts and rights and finally telling me I'd best be on my way if I wanted to reach town by sunset.Based on context clues in the passage, what is the most likely meaning of laconic? Select two options.talkativebriefopenwarmconcise If I start with 25.Og of Na and 15.0g of H2OAdd favoritea. how much H, gas will be produced?!bLR:ERC: Excess Left over: QUESTION 6 A reduced product of the Citric Acid Cycle is: O ATP O CO2 NADH O OAA